30 Neil's weight 510g. what is the mass of 48 Neil's​

Answers

Answer 1

at first..we need to find the value of one neil....

1 neil=510/10

=17

Therefore..one neil=17 g...

Now;

48 neil= 17×48

=816

Hence; The mass of 48 Neil's is 816...

hope this will help u


Related Questions


You have cup of cocoa in your cupboard. A recipe for cookies calls for 1
cup of cocoa. How much cocoa would you have left if you made the cookies?

Answers

Answer:

None

Step-by-step explanation:

sorry don't know how to delete this

Hi! I'm happy to help!

The recipe calls for 1/4 of a cup of cocoa, and we have 5/6 of a cup.

We use subtraction to solve for the difference:

5/6-1/4

To solve for this, we need to find a common denominator. We do this by finding the least common multiple of the two denominators we already have.

The least common multiple is 12, because that is the lowest number that both 6 and 4 are factors of.

We multiplied the 6 by 2 to get twelve, so, to make it equivalent, we have to multiply the 5 by 2 also.

10/12-1/4

Now, since we multiplied the 4 by 3 to get twelve, we need to do the same to the 1.

10/12-3/12

You never subtract denominators, only numerators. Now, all we have to do is 10-3:

10/12-3/12=

7/12

So, you would have 7/12 cup of cocoa left over if you made the cookies.

I hope this was helpful, keep learning! :D

PLSSSSSSSSSSSSSSSSs Help me i have to send this to my math teacher before yk back to school starts

Answers

Q/ True or False

4. Ans ; False —> because

[tex] \frac{2 \times 3.5}{3 \times 3.5} = \frac{7}{10.5} [/tex]

Or; 2/3= 0.6 , 7/16= 0.43 so no equal

5. True —> because

[tex] \frac{5 \times 9.6}{15 \times 9.6} = \frac{48}{144} [/tex]

Or; 5/15=0.33 , 48/144 = 0.33 so equal

6. True —> because

[tex] \frac{12 \times 1.5}{28 \times 1.5} = \frac{18}{42} [/tex]

Or; 12/28=0.4285 , 18/42=0.4285 so equal

Q/ value " x "

7. Ans;

[tex] \frac{3}{6} = \frac{x}{8} \\ 6x = 8 \times 3 \\ 6x = 24 \\ x = \frac{24}{6} \\ x = 4[/tex]

8. Ans;

[tex] \frac{52}{x} = \frac{4}{1} \\ 4x = 52 \\ x = \frac{52}{4} \\ x = 13[/tex]

9. Ans;

[tex] \frac{15}{12} = \frac{10}{x} \\ 15x = 10 \times 12 \\ 15x = 120 \\ x = \frac{120}{15} \\ x = 8[/tex]

10. Ans;

[tex] \frac{18}{x} = \frac{2.4}{28} \\ 2.4x = 28 \times 18 \\ 2.4x = 504 \\ x = \frac{504}{2.4} \\ x = 210[/tex]

11.Ans;

[tex] \frac{3}{4} = \frac{x}{3.8} \\ 4x = 3.8 \times 3 \\ 4x = 11.4 \\ x = \frac{11.4}{4} \\ x = 2.85[/tex]

12. Ans;

[tex] \frac{x}{12} = \frac{2 \frac{1}{3} }{5} \\ \frac{x}{12} = \frac{ \frac{7}{3} }{5} \\ \\ \frac{x}{12} = \frac{7}{3} \div 5 \\ \\ \frac{x}{12} = \frac{7}{3} \times \frac{1}{5} \\ \\ \frac{x}{12} = \frac{7}{15} \\ \\ 15x = 12 \times 7 \\ 15x = 84 \\ \\ x = \frac{84}{15} \\ \\ x = 5.6[/tex]

Q/ Write each ratio

1.Ans;

[tex] \frac{30 \: ft}{60 \: ft} = \frac{30 \: ft \: \: \div 30}{60 \: ft \: \: \div 30} = \frac{1 \: ft}{2 \: ft} [/tex]

2. Ans;

[tex] \frac{45 \: mph}{25 \: mph} = \frac{45 \: mph \: \div }{25 \: mph \div 5} = \frac{4 \: mph}{5 \: mph} [/tex]

3. Ans;

[tex] \frac{6 \: min}{16 \: min} = \frac{6 \: min \: \div2 }{16 \: min \: \div 2} = \frac{3 \: min}{8 \: min} [/tex]

4. Ans;

[tex] \frac{300}{450} = \frac{300 \div 150}{450 \div 150} = \frac{2}{3} [/tex]

5. Ans;

[tex] \frac{48 \: gal}{14 \: min} = \frac{48 \: gal \div 2}{14 \: min \div 2} = \frac{24 \: gal}{7 \: min} [/tex]

6. Ans;

[tex] \frac{21 \: fem}{61 \:mal } = \frac{21 \: fem \div 3}{51 \:mal \div 3} = \frac{7 \: fem}{17 \: mal} [/tex]

7. Ans;

[tex] \frac{4.2}{36} = \frac{4.2 \div 12}{36 \div 12 \: pot} = \frac{0.35}{3 \: pot} [/tex]

8. Ans;

[tex] \frac{72 \: correct}{96 \: questions} = \frac{72 \: correct \div 24}{96 \: questions \div 24} = \frac{3 \: correct}{4 \: questions} [/tex]

Q/Find the unit rate

9.Ans;

[tex] \frac{1500 \: m}{6 \: s} = \frac{250 \: m}{1 \: s} [/tex]

10. Ans;

[tex] \frac{192 \: mil}{6 \: gal} = \frac{32 \: min}{1gal} [/tex]

11. Ans;

[tex] \frac{20 \: people}{5 \: cars} = \frac{4 \: people}{1 \: cars}[/tex]

12.Ans;

[tex] \frac{36}{4 \: lbs} = \frac{9}{1 \: lbs} [/tex]

I hope I helped you^_^

(8.11×107)–(4.3×107)

Answers

Answer: 407.67

Explanation:

(8.11×107)–(4.3×107) =

(8.11)(107)−(4.3)(107) =

         407.67

49q²-70q+25 resolve into fractors​

Answers

Answer:

(7q-5)²

Step-by-step explanation:

= 49q² - 70q + 25

= 49q² -35q -35q + 25

= 7q(7q-5) - 5(7q - 5)

= (7q-5) (7q-5)

= (7q-5)²

CMIIW

Answer:

Step-by-step explanation:

49q² -70q +25 , rewrite the first two terms as squares

(7q)² -70q + 5² , compare with  a² - 2ab + b² = (a-b)²

   

    a² - 2ab + b²

(7q)² -2(7q)(5 )+ 5², because 70q = 2*35q = 2*7*5*q

We can see that a = 7q and b=5, therefore

49q² -70q +25 = (7q -5)²

We set the equation equal to 0 to find the roots

7q-5 =0, add 5 to both sides

7q =5 , divide both sides by 7

q = 5/7 ≈ 0.714

Use coordinate notation to enter the rule that maps each preimage to its image. Then ic
transformation and confirm that it preserves length and angle measure.
A(-5,5) ► A'(5,5)
B(-2,2) B'(2, 2)
C(-3,2) → C'(2,3)
The transformation is a rotation of ?
(x,y) →
° clockwise about the origin given by the rules
6
5

Answers

Coordinate geometry is the use of a 2D plane to represent points.

The transformation is a rotation of 270 degrees clockwiseThe transformation preserves length because the length of the image and preimage are the sameThe transformation preserves angles because the angles of the image and preimage are the same

Given that:

[tex]A = (-5,5) \to A' = (5,5)[/tex]

[tex]B = (-2,2) \to B' = (2, 2)[/tex]

[tex]C = (-3,2) \to C' = (2,3)[/tex]

By observing the pattern of transformation, the rule is:

[tex](x,y) \to (y,-x)[/tex]

Hence, the transformation is a rotation of 270 degrees clockwise

The length is calculated using the following distance formula:

[tex]d = \sqrt{(x_1 - x_2)^2 + (y_1 - y_2)^2[/tex]

So, we have:

[tex]AB = \sqrt{(-5 - -2)^2 + (5 - 2)^2} =\sqrt{18[/tex]

[tex]BC = \sqrt{(-2 - -3)^2 + (2 - 2)^2} =1[/tex]

[tex]AC = \sqrt{(-5 - -3)^2 + (5 - 2)^2} =\sqrt{13[/tex]

And

[tex]A'B' = \sqrt{(5 -2)^2 + (5 - 2)^2} =\sqrt{18[/tex]

[tex]B'C' = \sqrt{(2 - 2)^2 + (2 - 3)^2} =1[/tex]

[tex]A'C' = \sqrt{(5 - 2)^2 + (5 - 3)^2} =\sqrt{13[/tex]

By comparing the lengths of the image and the preimage, we can conclude that the transformation preserves length.

The measure of the angles is calculated as follows:

[tex]a^2 = b^2 + c^2 -2ab \cos A[/tex]

So, we have:

[tex]18 = 1 + 13 -2 \times 1 \times \sqrt{13} \cos C[/tex]

[tex]18 - 1 - 13= -2 \times 1 \times \sqrt{13} \cos C[/tex]

[tex]4= -2 \times 1 \times \sqrt{13} \cos C[/tex]

[tex]-2= \sqrt{13} \cos C[/tex]

Make cos C the subject

[tex]\cos C = -0.5547[/tex]

[tex]C = cos^{-1}(-0.5547)[/tex]

[tex]C = 124^o[/tex]

Also, we have:

[tex]\frac{a}{\sin A} =\frac{c}{\sin C}[/tex]

So, we have:

[tex]\frac{1}{\sin A} =\frac{\sqrt{18}}{\sin( 124)}[/tex]

[tex]\frac{1}{\sin A} =5.1175[/tex]

Rewrite as:

[tex]\sin A = \frac{1}{5.1175}[/tex]

[tex]\sin A = 0.1954[/tex]

[tex]A = \sin^{-1}(0.1954)[/tex]

[tex]A = 11^o[/tex]

Also, we have:

[tex]A + B + C = 180^o[/tex] --- sum of angles in a triangle

[tex]11 + B + 124 = 180^o[/tex]

Collect like terms

[tex]B = 180 -11 - 124[/tex]

[tex]B = 45[/tex]

Hence:

[tex]\angle A = 11[/tex]   [tex]\angle B = 45[/tex] and [tex]\angle C = 124[/tex]

Read more about coordinate geometry at:

https://brainly.com/question/1601567

your cell phone plan cost 24.99 per month plus $19 for each text message you send or receive you have at most $34 to spend on your cell phone bill what is the maximum number of text message you can receive or sent​

Answers

Answer:

1.

Step-by-step explanation:

You can't send two because it'd be 38

If 3 and 5 are factors of a number, then
15 is a factor of the number.

Answers

Answer:

factors of 15 are 3 and 5, because 3×5 = 15. Some numbers have more than one factorization (more than one way of being factored).

Step-by-step explanation:

The given statement, "If 3 and 5 are factors of a number, then

15 is a factor of the number", is true as the lowest common multiple of 3 and 5 is 15.

What do we mean by factors of a number?

A number x is said to be a factor of another number y when y/x leaves a remainder = 0.

How do we solve the given question?

In the question, we are asked that if 3 and 5 are factors of a number, then will 15 also be a factor of that number.

The smallest number with the factors 3 and 5 is 15, as the lowest common multiple of 3 and 5 is 15. Hence, the number we are looking for is a multiple of 15, that is, 15 is a factor of that number.

∴ We can say that any number having factors 3 and 5, will have a factor of 15. Hence, the given statement is true.

Learn more about factors of a number at

https://brainly.com/question/219464

#SPJ2

g(x)=x^2-x
g(-4)
help!

Answers

Answer:

g(-4) = 20

Step-by-step explanation:

g(x) = x^2 - x when x = -4

~Substitute

= (-4)^2 - (-4)

~Simplify

= 16 + 4

= 20

Best of Luck!

find the unknown angles

Answers

Answer:

Step-by-step explanation:

∠WZA + WZY = 180     {linear pair}

88 + ∠WZY = 180

∠WZY = 180 - 88

∠WZY = 92°

∠WXY + ∠WZY = 180     {Opposite angle in cyclic quadrilateral}

x + 92 = 180

       x = 180 - 92

x = 88°

what is the domain of the function

Answers

Answer:

(-∞,20}

Step-by-step explanation:

Use the commutative property to reorder the terms.

Separate the function into parts to determine the domain of each part

The domian of a rational function are all values of x which the denominator is different than 0

Find the intersection

how do you convert a repeating decimal in the 10000 place into a fraction?

Answers

Decimal to Fraction

Converting any terminating decimal into a fraction is fairly straightforward. You count the number of decimal places, move the decimal point that number of places to the right, and put the resulting number over "1" followed by that number of zeroes.

For instance:

Convert the decimal 0.7 into a fraction.

This decimal has one decimal place, so I move the decimal point one place to the right, to get 7. Then I put this over "1" followed by one zero (otherwise known as "10") to get: 7/10

Step-by-step explanation:

296 divide by 0.08 whats the answer plz answer, and show me how you did it plz

Answers

Answer:

Step-by-step explanation:

Ok so u can’t divide a decimal with a whole number so u have to move the decimal poin from the 8 twice so it’s on the other side of the 8 and then there shouldn’t be any zeros before the 8 now divide 296 by 8 and instead of leaving the whole number like that u have to add the decimal point to it. So it should look like this 296.00 and then u can divide which is 3,700 hope this helped <3

a) biswant thought of a number. he double it and added 5. if he got 13, find the number he thought​

Answers

Answer:

Four

Step-by-step explanation:

13- 5= 8

8/2=4

You simply do the problem in reverse.

Answer:

The number is 4

Step-by-step explanation:

Let the number = x

Double of x = 2*x = 2x

Then add 5 to it: 2x +5

2x +5 = 13

Subtract 5 form both side

2x + 5 - 5 = 13 - 5

         2x = 8

Divide  both sides by 2

       x = 8/2

x = 4

The line joining (3, p) to (7, –4p) is parallel to the line joining (–1, –3) to (3, 7). Find p.

Answers

Answer:

p = - 2

Step-by-step explanation:

Calculate the slope m using the slope formula

m = [tex]\frac{y_{2}-y_{1} }{x_{2}-x_{1} }[/tex]

with (x₁, y₁ ) = (- 1, - 3) and (x₂, y₂ ) = (3, 7)

m = [tex]\frac{7-(-3)}{3-(-1)}[/tex] = [tex]\frac{7+3}{3+1}[/tex] = [tex]\frac{10}{4}[/tex] = [tex]\frac{5}{2}[/tex]

Parallel lines have equal slopes

Calculate the slope of the other 2 points and equate to [tex]\frac{5}{2}[/tex]

with (x₁, y₁ ) = (3, p) and (x₂, y₂ ) = (7, - 4p)

m = [tex]\frac{-4p-p}{7-3}[/tex] = [tex]\frac{-5p}{4}[/tex] = [tex]\frac{5}{2}[/tex] ( cross- multiply )

- 10p = 20 ( divide both sides by - 10 )

p = - 2

(NEED HELP PLS ANSWER GIVING 100 POINTS) Following my question of "If a student didn't turn an assignment in on the 27th of august, and the teacher deducted 10 percent each day. How much credit would they get if they turned it in on the 1st of September?" If the students current grade was a 41 F how higher would it get if they turned it in ( worth 40% of grade)

Answers

Answer: The student would get 70% credit.

The student’s overall grade would be

Step-by-step explanation:

Consider a triangle ABC like the one below. Suppose that A=121° C = 25°, and a=64. (The figure is not drawn to scale.) Solve the triangle.
Round your answers to the nearest tenth.
If there is more than one solution, use the button labeled "or".

Answers

Answer:

4

Step-by-step explanation:

i need points bro

Need help with this math question. Will give brainliest

Answers

Reason 4 is incorrect. He is multiplying both sides, not dividing.

A pound of chocolate costs 7 dollars. David buys p pounds. Write an equation to represent the total cost c that David pays.

Answers

Answer:

C = 7p

Step-by-step explanation:

Each pound costs 7 dollars, and he buys P, an unknown amount. This equals C.

How do I explain this??

Answers

Answer:

No

Step-by-step explanation:

The x-axis shows Craig's pitch speed in miles per hour.

The highest speed he did was between 70 to 75 mph. Craig's doesn't specify the speed in the histogram, so we can only be sure that Craig's fastest is between 70 to 75 mph.

Your job is 7.5 miles from home. One option is to take the back roads to work. That route is calculated for driving at an average speed of 30 miles per hour. How long will it take you to get to work this way?

Answers

Step-by-step explanation:

So if you are 7 and a half miles away, and you drive 30 miles per hour, you need to take 7.5 and divide it by it by 30 and you get 0.25, you then need to find what is a quarter (0.25) of 60 minutes, and you will find that it is 15 minutes.

So:

Answer:

The route will take 15 minutes to get to work taking the back roads.

Hope this helps!

which figures have rotation Cemetery​

Answers

The first diamond shaped one and the arrows!

Have a nice day :D

Need help with # 12 for distance/ midpoints formula. And another problem for #1 distance, segment and radicals ? Please help

Answers

Step-by-step explanation:

#12

PQ = 6x + 25

QR = 16 - 3x

PR = 6x + 25 + 16 - 3x

PR = 3x + 41

#1

D = √(-4 - 5)² + (-7 - 2)²

D = √(-9)² + (-9)²

D = √81 + 81

D = √162

D = 9√2

The formula Q = MCT we're Q = heat flow, M = mass, C = Specific heat, and T = change of temperature is used to calculate heat flow. Solve this formula for T.

Answers

the answer must be t equal to Q over MC

Answer:

T= Q over MC

Step-by-step explanation:

I did the test

If Σfx = 60+45a and Σf = 12+9a , find the mean.

Answers

Step-by-step explanation:

Σfx = 60+45a

Σf = 12+9a

Now

Mean =Σfx/Σf

= 60+45a/12+9a

=105/21 (a has been canceled)

=5

Therefore mean =5

HELP PLEEAASSEEE!!!!!!!
Drag the tiles to the correct boxes to complete the pairs.
Match the numbers in standard notation with the corresponding numbers in scientific notation.

Answers

8.64E9 = 8640,000,000
8.64E14 = 864,000,000,000,000
8.64E11 = 864,000,000,000
8.64E13 = 86,400,000,000,000

Indicate the method you would use to prove the two A's =. If no method applies, enter "none".
O ASA
O AAS
A
• None

SSS
O SAS

Answers

Answer:

SSS is used

Step-by-step explanation:

mark me as brainliest ❤️

The triangles are congruent with SSS postulates. Then the correct option is D.

What is the triangle?

The polygonal shape of a triangle has a number of sides and three independent variables. Angles in the triangle add up to 180°.

Two triangles are shown.

The SSS postulate is one of the ways to prove that two triangles are congruent. SSS stands for "side-side-side," and it states that if three sides of one triangle are congruent to the corresponding sides of another triangle, then the two triangles are congruent.

The corresponding sides of the triangle will be equal. Then the triangles are congruent with SSS postulates.

Thus, the correct option is D.

More about the triangle link is given below.

https://brainly.com/question/25813512

#SPJ7

Rewrite in the simplest terms: 8k - 9(4k +4)

Answers

8k - 9(4k + 4) = 8k - 36k - 36 = -28k - 36

Let's rewrite in the simplest term,

→ 8k - 9(4k + 4)

→ 8k - 36k - 36

→ [-28k - 36]

The simplest term is -28k - 36.

The simple interest on a sum of money in 2 years is Rs 90 less than its compound interest. If the rate of interest in a year is 15%, find the sum.

Answers

Answer:

hence, the req sum of amount is Rs 213.07

check both the files . and got any confusion then comment it .

hope this helped you

Solve the equation
6x + 8 = 44
What is the solution set? Select the correct choice below and, if necessary, fill in the answer box within your choice.

Answers

Answer:

x = 6

Step-by-step explanation:

6x = 36

x = 6

hope that helps

Answer:

6

Step-by-step explanation:

6x + 8 = 44

6x = 44 - 8

6x = 36

x = 6

therefore the right answer is 6

What is the product of 5s and (-52 +38 – 9)?

Answers

Answer:

1st one

Step-by-step explanation:

it is by multiple of 5 withthe words inside bracket.

Answer:

52

+38

=90

-9

=81

so the answer is=81

Other Questions
Select the choice that translates the following verbal phrase correctly to algebra:the difference of m and 7 increased by 15 m (7 + 15) 7m + 15 (m 7) + 15 m 7 15 Find cos(x) if [tex]\frac{sin^2x-1}{cosx} =-1[/tex].A -1B 2C 1D 0 Read the excerpt from part two of Trifles. Which idea is suggested by this excerpt? Explain how you know that the sums of 12.6,3.1 and 5.4 is greater than 20 You would be enjoying a coastline if you were walking along:A. A sloping mound of earthB. A narrow strip of land that ends by a riverC. A stretch of land along the ocean or seaD. Higher ground made up of mountains 11 4/5 + -8 1/3NO LINKS OR TROLLING How are the basic economic questions decided in a traditional economy?O A They are deg med based on custom and habit.B. They are decided based on currency exchange.O c. They are decided based on government priorities.D. They are decided based on consumer preferences. What is 4=2x as a improper fraction or integer Which of the following is equal to the rational expression when x -1 or -7?(X+1)(x-1)(X+7)(x+1) A. x+1 x+7B. x+7 x-1C. x-1 x+7 Evaluate x(6)2When x = -4 What was Spain's response to the claims of other European countries in North America John wants to buy strawberries and apples to make a fruit tart. Strawberries cost $3 per pound and apples cost $2.25 per pound. How much does he spend if he buys 0.5 pounds of strawberries and 3 pounds of apples? How much does he spend if he buys xx pounds of strawberries and yy pounds of apples? oi oi, so I'm taking medicines with my aunt (she is a paramedic) but I don't understand the codes like code red, code blue, I only know code black is a bomb.. what are the different codes for? What is the area of the shaded rectangle? QUESTIONSlan has $20 in savings. Hedecides to spend 4 of it. Howmuch will he have left? Discuss the intervention measures to mitigate the effects of overstocking to improve performance of the indigeous cattle. Pa help naman po please Find the value of x50x which of the following is a reason people might move? For flocabulary . A truck travels 40 m while slowing down from 15 m/s to 10 m/s. How long does it take to do this?